Difference between revisions of "2014 AMC 10B Problems/Problem 6"

(See Also)
(Problem)
Line 1: Line 1:
==Problem==
+
==Problem 6==
 +
 
 +
Orvin went to the store with just enough money to buy <math>30</math> balloons. When he arrived, he discovered that the store had a special sale on balloons: buy <math>1</math> balloon at the regular price and get a second at <math>\frac{1}{3}</math> off the regular price. What is the greatest number of balloons Orvin could buy?
 +
 
 +
<math> \textbf {(A) } 33 \qquad \textbf {(B) } 34 \qquad \textbf {(C) } 36 \qquad \textbf {(D) } 38 \qquad \textbf {(E) } 39</math>
  
 
==Solution==
 
==Solution==

Revision as of 12:49, 20 February 2014

Problem 6

Orvin went to the store with just enough money to buy $30$ balloons. When he arrived, he discovered that the store had a special sale on balloons: buy $1$ balloon at the regular price and get a second at $\frac{1}{3}$ off the regular price. What is the greatest number of balloons Orvin could buy?

$\textbf {(A) } 33 \qquad \textbf {(B) } 34 \qquad \textbf {(C) } 36 \qquad \textbf {(D) } 38 \qquad \textbf {(E) } 39$

Solution

See Also

2014 AMC 10B (ProblemsAnswer KeyResources)
Preceded by
Problem 5
Followed by
Problem 7
1 2 3 4 5 6 7 8 9 10 11 12 13 14 15 16 17 18 19 20 21 22 23 24 25
All AMC 10 Problems and Solutions

The problems on this page are copyrighted by the Mathematical Association of America's American Mathematics Competitions. AMC logo.png